Pre-trial considerations MCQs Flashcards

1
Q

You represent Kaseen at the police station for an allegation of robbery. The police tell him that they plan to charge him with the offence and he asks for your advice regarding which court his case will be heard in.

What is the best advice that you can give to Kaseem regarding his case?

His first appearance and all future hearings will take place in the Crown Court because robbery is an indictable only offence.

His first appearance will take place in the Crown Court but he may have future hearings in the magistrates’ court depending on the seriousness of the robbery as less serious robberies are either-way offences

His first appearance will take place in the magsitrates’ court but he will then be sent straight to the Crown Court because robbery is an indictable only offence.

His first appearance will take place in the magistrates’ court and he will have the choice as to where any future hearings take place as robbery is an either-way offence.

A

His first appearance will take place in the magsitrates’ court but he will then be sent straight to the Crown Court because robbery is an indictable only offence.

Correct – robbery is an indictable only offence so his case will start in the magistrates’ court but all future hearings have to take place in the Crown Court.

How well did you know this?
1
Not at all
2
3
4
5
Perfectly
2
Q

You represent Dan, a teacher who has been arrested for an offence of common assault against one of his pupils. It is alleged that the pupil was shouting out in class and causing a nuisance, so Dan ordered him to leave the classroom. When the pupil refused to leave the room, it is alleged that Dan grabbed him by the harm and physically removed him from the classroom.

Dan denies the allegation stating that he merely moved towards the pupil and followed him as he left the room but did not make any contact with him.

Dan earns £28,000 per year and has savings of around £20,000 which he has been planning to use to buy his first house.

Which of the following correctly sets out the position regarding funding for Dan’s case?

Dan will be entitled to free legal advice at the police station and pass the merits test for legal aid at the magistrates’ court but will likely fail the means test due to his earnings and savings

Dan will be entitled to free legal advice at the police station but is unlikely to pass the merits or the means test for legal aid at the magsitrates’ court because the offence is not serious enough and his earnings and savings are too high.

Dan will not be entitled to free legal advice at the police station or legal aid at the magistrates’ court because he is likely to fail the means test due to his earnings and savings.

Dan will not be entitled to free legal advice at the police station or legal aid at the magistrates’ court because the offence is not serious enough to pass the merits test.

A

Dan will be entitled to free legal advice at the police station and pass the merits test for legal aid at the magistrates’ court but will likely fail the means test due to his earnings and savings

Correct – everyone is entitled to free legal advice at the police station irrespective of their income and savings. Dan would certainly pass the merits test due to the nautre of the allegation and the risk to his livelihood and reputation if he were to be convicted but he will most likely be ineligible for legal aid because his earnings and savings are too high.

How well did you know this?
1
Not at all
2
3
4
5
Perfectly
3
Q

You represent Cheng, a 27 year old, who is to be sentenced in the Crown Court for offences of theft. The pre-sentence report confirms that Cheng has an addiction to drugs that contributed to his decision to commit the offences. He pleaded guilty to the offences and has indicated a willingness to take steps to address the addiction.

Having regard to the statutory purposes of sentencing, which of the following statements is the most accurate?

The court should require Cheng to address his addiction under the terms of a community order with a drug rehabilitation requirement because the principal aim of the justice system is to prevent offending (or re-offending).

The court could impose a community order with a drug rehabilitation requirement because the court may have regard to the reform and rehabilitation of offenders when considering what sentence to impose.

The court could only impose a community order with a drug rehabilitation requirement if it is satisfied that this will adequately protect the public because the protection of the public is the paramount consideration when passing sentence.

The court could consider making a community order with a drug rehabilitation requirement requiring Cheng to address his addiction because the reform and rehabilitation of offenders is one of the purposes of sentencing which the court must have regard to.

A

The court could consider making a community order with a drug rehabilitation requirement requiring Cheng to address his addiction because the reform and rehabilitation of offenders is one of the purposes of sentencing which the court must have regard to.

This is correct. S.57 Sentencing Act 2020 sets out the purposes of sentencing adults. S.57(2) sets out 5 purposes which the court must have regard to. The reform and rehabilitation of offenders is one of those purposes.

How well did you know this?
1
Not at all
2
3
4
5
Perfectly
4
Q

You are representing Millie in the magistrates’ court for an offence of common assault. It is alleged that she slapped her friend Meghan around the face. Millie tells you that she doesn’t think Meghan will turn up to court so wants to plead not-guilty.

She asks you not to complete the case management form as she doesn’t want the court to know that this is her plan.

What response should you give to Millie?

You have an obligation to complete the case management form but you can still allow Millie to enter a not guilty plea and put the prosecution to proof.

You have an obligation to complete the case management form and you will have to advise Millie that you are not able to represent her unless she has a positive defence to advance.

You do not have an obligation to complete the case management form so could follow Millie’s instructions regarding this and her plan to plead not guilty and put the prosecution to proof.

You do not have an obligation to complete the case management form so could follow Millie’s instructions regarding this but you are not able to allow Millie to plead not guilty just to put the prosecution to proof.

A

You have an obligation to complete the case management form but you can still allow Millie to enter a not guilty plea and put the prosecution to proof.

Correct – you have an obligation to comply with the overriding objective and take an active role in case management and this includes completing the case management form. You could allow Millie to enter a not guilty plea and put the prosecution to proof but would need to weigh up if this is in your client’s best interests based on the evidence against her

How well did you know this?
1
Not at all
2
3
4
5
Perfectly
5
Q

Ronald has been arrested for robbery. It is alleged that he attacked an elderly woman in the street at knife point and stole her handbag with her wallet, house keys and pension book inside it. He was arrested at his home address following a tip off from a member of the public. The pension book was subsequently found in a search of his home address. He was identified by the victim at a video identification parade. Following a Police National Computer check, it was discovered that Ronald has one previous conviction for a street robbery and an outstanding warrant for his arrest for an alleged assault.

What is the best explanation for the police duty of pre-interview disclosure in this case?

Ronald must be made aware of the nature of the suspected offence.

Ronald must be told he was identified by the victim at a video identification parade.

Ronald must be told the pension book was found in his home.

Ronald must be given sufficient information to enable him to understand the nature of the offence and why he is suspected of committing it.

A

Ronald must be given sufficient information to enable him to understand the nature of the offence and why he is suspected of committing it.

This is the best answer. It sets out the police duty of disclosure under PACE Code C, para. 11.1A.
See your materials on preliminaries to prosecution, Code C.11 (interviews) for further details.
Ronald merely being made aware of the nature of the suspected offence, does not accurately reflect the police duty of disclosure. The decision on what should be disclosed to Ronald (e.g. finding the pension book or being identified by the victim) rests with the investigating officer.

How well did you know this?
1
Not at all
2
3
4
5
Perfectly
6
Q

PC Hall and PC Patel receive a call that a burglary has just taken place at a local address where a television and some jewellery was stolen. The three suspects are described as white males, between 20-25 years of age, wearing black hoodies and blue jeans; two males have short, dark hair and one has blonde curly hair. PC Hall spots three males matching the description carrying a television, two streets away from where the burglary took place. The males are arrested on suspicion of burglary, cautioned and taken to the Police Station. In response to the caution on arrest one of the suspects says ‘we are just taking the television from my grandmother’s house to my mother’s.’

Which of the following best describes the procedure that the police should follow during the interview?

The suspects should be cautioned and offered legal advice before the officers commence questioning regarding the burglary and the reason they had the television in their possession.

The suspects should be cautioned, offered legal advice, issued with the special warning at the start of the interview and asked regarding the comment that the suspect made on arrest at the relevant point during the questioning.

The suspects should be cautioned, offered legal advice and issued with the special warning at the beginning of the interview before commencing questioning regarding the burglary and the reason they had the television in their possession.

The suspects should be cautioned, offered legal advice, asked regarding the comment made on arrest at the start of the interview and given the special warning before asking questions regarding having the television in their possession.

A

The suspects should be cautioned, offered legal advice, asked regarding the comment made on arrest at the start of the interview and given the special warning before asking questions regarding having the television in their possession.

This is the correct answer as it has all the requirements in the correct order for the interview and all questions to be lawful, avoiding a challenge for admissibility.
A special warning should be given but only when the suspects are questioned regarding their possession of the television or their location near the scene.
The significant comment made by one of the suspects on arrest must be put to them at the start of the interview not at any stage during the questioning.

How well did you know this?
1
Not at all
2
3
4
5
Perfectly
7
Q

PC Hall arrests Jackie following her suspected involvement in a fight that took place in a pub two nights ago.

What reason should PC Hall give for the arrest?

PC Hall has reasonable grounds to suspect that Jackie has committed an offence and to allow for the prompt and effective investigation of the offence.

PC Hall has reasonable grounds to suspect that Jackie has committed an arrestable offence and needs to question Jackie about her involvement in that offence.

PC Hall has reasonable grounds to believe that Jackie has committed an offence and arrested Jackie in order to prevent physical injury being caused to herself or others.

PC Hall has reasonable grounds to believe that Jackie has committed an indictable offence and needs to question Jackie about her involvement in that offence.

A

PC Hall has reasonable grounds to suspect that Jackie has committed an offence and to allow for the prompt and effective investigation of the offence.

Correct. PC Hall has reasonable grounds to believe that Jackie has committed that offence and the reason given is one of the reasons set out in s.24(5) PACE.
Whilst the other options sound plausible, they are not the best answer. There must be a reason for the arrest set out in s.24(5) PACE.
Questioning about Jackie’s involvement is not expressly one of the reasons set out in s.24(5) PACE; questioning about her involvement would come under allowing prompt and effective investigation of the offence s.24(5)(e) PACE.
The offence does not need to be an indictable offence in order for PC Hall to make the arrest.
Preventing physical injury being caused to herself or another is one of the reasons for arrest as set out in s.24(5)(c). However, as we are told on the facts that the fight took place two nights ago, it is unlikely that Jackie is still a risk of causing injury to herself or another at the point of this arrest being made.

How well did you know this?
1
Not at all
2
3
4
5
Perfectly
8
Q

Dennis has been arrested for possession of a controlled drug (cocaine) with intent to supply, which is an indictable offence. His detention without charge has been extended by an officer of the rank of superintendent.

What is the maximum number of hours that Dennis can be detained for?

Dennis can be detained for a maximum of 48 hours

Dennis can be detained for a maximum of 36 hours

Dennis can be detained for a maximum of 24 hours

Dennis can be detained for a maximum of 96 hours

A

Dennis can be detained for a maximum of 36 hours

This is the correct answer. The basic 24 hours detention without charge can be extended up to 36 hours from the ‘relevant time’ for an indictable offence (which includes an either-way offence such as possession of a class A drug with intent to supply) where s.42(1) PACE 1984 applies: * further detention is authorised by an officer who holds the rank of superintendent or above; and * the officer has reasonable grounds for believing that it is necessary to detain Dennis without charge to secure or preserve evidence or to obtain evidence by questioning him; and * the investigation is being conducted diligently and expeditiously.
96 hours is the maximum period of detention without charge that can be authorised by a magistrates’ court if they find there are reasonable grounds for believing that further detention is justified.

How well did you know this?
1
Not at all
2
3
4
5
Perfectly
9
Q

Ryan is charged with an offence of criminal damage. The allegation is that he threw stones at stained glass windows in a new community centre causing £6,000 of damage. Your instructing solicitor asks whether there is a time limit within which the prosecution must begin his case.

What advice will you give your instructing solicitor?

The prosecution needs to commence proceedings within 12 months of the date of the alleged offence

The prosecution needs to commence proceedings within 6 months of the date of the alleged offence

The prosecution has no time limit within which it needs to commence proceedings

The prosecution needs to commence proceedings within 6 years of the date of the alleged offence

A

The prosecution has no time limit within which it needs to commence proceedings

Correct. There is no time limit for commencing proceedings in indictable offence cases and criminal damage over £5,000 is an either-way offence.
The prosecution needs to commence proceedings within 6 months of the date of the alleged offence in summary only cases.
The timescales of 12 months and 6 years relate to various civil law limitation periods, rather than the time limits for commencing proceedings in criminal cases.

How well did you know this?
1
Not at all
2
3
4
5
Perfectly
10
Q

Eddie is charged with robbery. He is kept in custody by the police and appears in the magistrates’ court the following day.

Which of the following items is Eddie entitled to receive as Initial Details of the Prosecution Case?

Eddie is entitled to a list of his previous convictions

Eddie is entitled to a summary of the circumstances of the offence

Eddie is entitled the prosecution witness statements and a list of his previous convictions

Eddie is entitled to a summary of the circumstances of the offence and a list of his previous convictions

A

Eddie is entitled to a summary of the circumstances of the offence and a list of his previous convictions

Eddie is a ‘custody overnight’ case as he was in police custody immediately before the first hearing in the magistrates’ court. Pursuant to CrimPR 8.3 Eddie is entitled only to a summary of the circumstances of the offence and a list of previous convictions.
All of the other options are not the best answer as:
· they are incomplete- Eddie is entitled to both a summary of the circumstances of the offence and a list of his previous convictions; and
· incorrect- Eddie is not entitled to the prosecution witness statements at this stage.
Note that if Eddie was not in custody overnight he would be entitled to any written witness statement or exhibit that the prosecutor has available and considers material to plea, allocation for trial or sentence.

How well did you know this?
1
Not at all
2
3
4
5
Perfectly
11
Q

Wendy is charged with criminal damage to a car and the value of the damage is £6,000.00. She will plead not guilty.

Where Wendy will be tried?

She may be tried at a magistrates’ court, but the magistrates can only sentence her to a maximum of three months’ imprisonment.

She must be tried at the Crown Court.

She may be tried at a magistrates’ court, but the magistrates can only sentence her to a maximum of six months imprisonment.

She must be tried at a magistrates’ court.

A

She may be tried at a magistrates’ court, but the magistrates can only sentence her to a maximum of six months imprisonment.

Correct. Since the value of the criminal damage is £6,000, it exceeds the ‘relevant sum’ (£5,000 at present) and renders it an either-way offence. Magistrates ‘may’ try Wendy’s case if after mode of trial the case is suitable for summary trial and the defendant consents to summary trial. As an either way offence the maximum sentence in a magistrates’ court is six months’ imprisonment and/ or a fine.
The other options were incorrect because:
· indictable only cases must be tried at the Crown Court and this is not an indictable only case; and
· summary only cases must be tried at a magistrates’ court and this is not a summary only case.

How well did you know this?
1
Not at all
2
3
4
5
Perfectly
12
Q

You prosecute a case in which 2 defendants, Paul and Emma are jointly charged with theft. At the first hearing they both indicate not guilty pleas.

Which of the following statements best describes the next step that will be followed?

The Magistrates will consider whether the trial can take place in the Magistrates’ Court. If they decide it can then Paul and Emma will each be asked if they consent. If one Defendant consents to summary trial then the other Defendant’s trial must also take place in the Magistrates’ Court.

The Magistrates will consider whether the trial can take place in the Magistrates’ Court. If they decide it can then Paul and Emma will each be asked if they consent. If Paul elects Crown Court trial and Emma consents to summary trial then Paul will be committed to Crown Court for trial and Emma’s case will be adjourned in the Magistrates’ Court to a date after Paul’s trial has concluded.

The Magistrates will consider whether the trial can take place in the Magistrates’ Court. If they decide it can then Paul and Emma will each be asked if they consent. The trial may only take place in the Magistrates’ Court if both consent.

The Magistrates will consider whether the trial can take place in the Magistrates’ Court. If they decide it can then Paul and Emma will each be asked if they consent. If Paul elects Crown Court trial and Emma consents to summary trial then there will be two separate trials because each Defendant has the right to choose where to be tried.

A

The Magistrates will consider whether the trial can take place in the Magistrates’ Court. If they decide it can then Paul and Emma will each be asked if they consent. The trial may only take place in the Magistrates’ Court if both consent.

This is correct. If one Defendant elects Crown Court trial then the Court must commit all other Defendants charged with the same offence if they are appearing at the Magistrates’ Court on the same occasion (S.51(5) Crime and Disorder Act 1988).

How well did you know this?
1
Not at all
2
3
4
5
Perfectly
13
Q

Nathan has been charged with causing grievous bodily harm with intent contrary to S.18 Offences Against The Person Act 1861. The custody officer has refused bail and told Nathan that he will appear in court in custody tomorrow.

Which of the following statements is correct in relation to tomorrow’s hearing?

The hearing will take place at the Crown Court.

The hearing will take place at the Magistrates’ Court. The Magistrates will ask Nathan to indicate a plea. If he indicates a guilty plea they will proceed to consider sentence. If he indicates a not guilty plea they will consider whether his trial should take place in the Magistrates’ Court or the Crown Court.

The hearing will take place at the Magistrates’ Court. The Magistrates will then send the case to the Crown Court but cannot consider bailing the Defendant as that decision is reserved to a Crown Court Judge in such cases.

The hearing will take place in the Magistrates’ Court. The Magistrates will then send the case to the Crown Court. They will also consider the issue of bail.

A

The hearing will take place in the Magistrates’ Court. The Magistrates will then send the case to the Crown Court. They will also consider the issue of bail.

This is correct. The first hearing in all cases is in the Magistrates’ Court regardless of the classification of offence. This offence is indictable only so the case must then be sent to the Crown Court for trial but the Magistrates are empowered to deal with ancillary matters such as the grant of bail at the first hearing.

How well did you know this?
1
Not at all
2
3
4
5
Perfectly
14
Q

Which of the following correctly explains the rules on service of a defence statement?

In the Crown Court, a defence statement must be served within 28 days of the prosecution complying, or purporting to comply, with its duty of initial disclosure of unused material.

In the magistrates’ court, a defence statement must be served within 28 days of the prosecution complying, or purporting to comply, with its duty of initial disclosure of unused material.

In the Crown Court, a defence statement must be served within 28 days of the prosecution complying, or purporting to comply, with its duty of initial disclosure of unused material. A failure to do so allows the court to draw such inferences as appear proper in deciding whether the defendant is guilty.

In the magistrates’ court, a defence statement must be served within 14 days of the prosecution complying, or purporting to comply, with its duty of initial disclosure of unused material. A failure to do will allow the court to draw an adverse inference.

A

In the Crown Court, a defence statement must be served within 28 days of the prosecution complying, or purporting to comply, with its duty of initial disclosure of unused material. A failure to do so allows the court to draw such inferences as appear proper in deciding whether the defendant is guilty.

Correct. In the Crown Court, a defence statement is compulsory, so once initial disclosure of unused material has been made, a defence statement must be filed and served by the defence within 28 days. Under section 11 of the 1996 Act, if a defendant fails to disclose at all or fails to do so within the time limit, or at trial, relies on a defence, or facts, not mentioned in the defence statement including alibi or witnesses the court may draw an inference
In the magistrates’ court a defence statement is not compulsory, but if a defendant chooses to serve a defence statement, standard directions provide the defendant must do so within 14 days of the prosecution complying, or purporting to comply, with its initial duty of disclosure.

How well did you know this?
1
Not at all
2
3
4
5
Perfectly
15
Q

The defendant is charged with an offence of assault occasioning actual bodily harm (‘ABH’) contrary to s. 47 Offences Against the Person Act 1861. They have entered a ‘not guilty’ plea and his case has been adjourned for a summary trial. You have received the initial details of the prosecution case (‘IDPC’). The defendant has instructed you not to serve a defence statement.

Which of the following best reflects the position regarding the prosecution’s obligation to serve unused material?

The prosecutor has a duty to disclose any prosecution material which might undermine the prosecution case or assist the case for the defence. Therefore, the defendant is entitled to disclosure of any material which meets this test.

In order to have unused material disclosed it will be necessary to make an application to the court for disclosure under the Criminal Procedure and Investigations Act 1996

In order to have unused material disclosed it is necessary to request the material on the defence statement. The prosecutor will then decide whether the material might undermine the prosecution case or assist the case for the defence

The prosecutor has a duty under the Criminal Procedure and Investigations Act 1996 to disclose any prosecution material which might undermine the prosecution case or assist the case for the defence. However, this applies only after the defendant has served a defence statement.

A

The prosecutor has a duty to disclose any prosecution material which might undermine the prosecution case or assist the case for the defence. Therefore, the defendant is entitled to disclosure of any material which meets this test.

Well done. The Criminal Procedure and Investigations Act 1996 imposes a duty on the prosecution to review the material and disclose any prosecution material which might undermine the prosecution case or assist the case for the defence. This applies whether the case is being tried in the magistrates’ or Crown Court and the duty to disclose commences before the defence statement is served when the defendant pleads not guilty in the magistrates’ court.
Applications can be made to the court but are only necessary if the defence believe the prosecution has not disclosed material in accordance with its duty.

How well did you know this?
1
Not at all
2
3
4
5
Perfectly
16
Q

You represent Mya who has been charged with s.18 OAPA. You notice a reference to CCTV on the schedule of unused material which is marked as capable of undermining the prosecution case.

Which of the following correctly sets out the position regarding disclosure of the CCTV?

You should make an application for disclosure under s.8 Criminal Procedure and Investigation Act 1996 in order to make the prosecution serve the CCTv. You do not need to serve a defence statement.

You do not need to serve a defence statement or make an application as the prosecution must disclose the CCTV as part of the initial disclosure.

You should serve a defence statement and then make an application for disclosure under s.8 Criminal Procedure and Investigation Act 1996 in order to make the prosecution serve the CCTV

You do not have any right to have the CCTV disclosed as it is part of the schedule of unused material

A

You should serve a defence statement and then make an application for disclosure under s.8 Criminal Procedure and Investigation Act 1996 in order to make the prosecution serve the CCTV

Correct- a defence statement must be served in order to make an application for specific disclosure under s.8 CPIA 1996.

How well did you know this?
1
Not at all
2
3
4
5
Perfectly
17
Q

You represent Aman who is charged with burglary and his case has been sent to the Crown Court for trial. He is required to serve a defence statement.

Which ONE of the following is NOT something that you would include in the defence statement?

Details of the discussions he has had with his solicitors prior to his police interview.

The matters of fact on which he intends to rely for the purposes of his defence.

The matters of fact on which he takes issue with the prosecution case and why.

The nature of his defence including any particular defences he is seeking to rely on.

A

Details of the discussions he has had with his solicitors prior to his police interview.

Correct – this information should not be included in the defence statement as is covered by privilege.

How well did you know this?
1
Not at all
2
3
4
5
Perfectly
18
Q

You are representing Mark for an offence of murder. It is alleged that Mark has a connection to a terrorist organisation and the murder was carried out as part of a terrorist attack. The prosecution has declined to disclose certain information and cited public interest immunity as the reason.

Which of the following correctly sets out the position regarding the prosecutions decision to refuse disclosure?

The information can only be withheld if a court orders that it is protected by public interest immunity.

The information can be withheld if the prosecution deem that it is protected by public interest immunity.

The information can be disclosed and the defence should decide if it is covered by public interest immunity.

The information should never be disclosed where there is a suggestion of public interest immunity

A

The information can only be withheld if a court orders that it is protected by public interest immunity.

Correct – it is for the court to determine whether information should be withheld for reasons of public interest immunity.

How well did you know this?
1
Not at all
2
3
4
5
Perfectly
19
Q

Your client attends your office in advance of his Plea and Trial Preparation Hearing (PTPH) at the Crown Court. He is facing six charges of burglary to which he will plead not guilty. He has various questions about what will happen with his case. In particular, he wants to know whether his previous convictions will be admitted as evidence, whether one of the witnesses who is only 13 years old will be allowed to give evidence via videolink, and what will happen in relation to his police interview during which officers repeatedly stated he was ‘talking nonsense’.

Which of these statements is the most accurate summary of what will happen at the PTPH in relation to your client’s questions?

A timetable will be set for the prosecution to make a bad character application and the defence reply. The court will hear argument from prosecution and defence as to whether the 13 year old witness should be granted special measures. The prosecution will be given a timetable by which to serve a ‘final’ edited version of the police interview transcript.

A timetable will be set for the prosecution to make a bad character application and the defence reply. The court will make directions as to special measures for the 13 year old witness, who automatically qualifies for special measures. The court will set a timetable for the prosecution and defence to agree an edited version of the police interview transcript.

A timetable will be set for the prosecution to make a bad character application and the defence reply. The court will hear argument from prosecution and defence as to whether the 13 year old witness should be granted special measures. The court will set a timetable for the prosecution and defence to agree an edited version of the police interview transcript.

The prosecution will notify the court that they will be making a bad character application at trial. The court will make directions as to special measures for the 13 year old witness, who automatically qualifies for special measures. The court will set a timetable for the prosecution and defence to agree an edited version of the police interview transcript.

A

A timetable will be set for the prosecution to make a bad character application and the defence reply. The court will make directions as to special measures for the 13 year old witness, who automatically qualifies for special measures. The court will set a timetable for the prosecution and defence to agree an edited version of the police interview transcript.

Correct. Timetables will be set for bad character applications as well as agreement of interview transcript. Because the witness is under 18 she automatically qualifies for special measures.
The other answers while plausible are incorrect:
Bad character applications can not be made at trial. An application must be served and responded to as per the court’s standard directions. At a later hearing argument will be heard as to bad character.

How well did you know this?
1
Not at all
2
3
4
5
Perfectly
20
Q

The defendant indicated a not guilty plea to a section 20 GBH at a plea before venue hearing and the matter is sent to Crown Court for trial.

Which of the following best explains what will happen next?

The next hearing will be the trial in the Crown Court. As it is an either way offence the magistrates court will be able to deal with all pre-trial matters including witness requirements and provide a timetable for the necessary pre-trial preparation and give appropriate directions for an effective trial

Unless it is a complex case the Pre-trial matters can be dealt with by way of standard directions in writing once the PTPH form has been submitted.

The next hearing will be the Plea and Trial Preparation Hearing which should take place within 28 days. The defendant will be arraigned, and the matter will be allocated a trial date.
Once the defendant pleads not guilty this will trigger the prosecution duty of disclosure

The next hearing will be the Plea and Trial Preparation Hearing which should take place within 28 days. The defendant will be arraigned, and the matter will be allocated a trial date.

A

The next hearing will be the Plea and Trial Preparation Hearing which should take place within 28 days. The defendant will be arraigned, and the matter will be allocated a trial date.

Well done.
The next hearing will be the Plea and Trial Preparation Hearing the court will:​
Set the trial date ​
Identify, so far as can be determined at that stage, the issues for trial
Consider with the parties the witness requirements that can be determined at that stage​
Provide a timetable for the necessary pre-trial preparation and give appropriate directions for an effective trial
Although the parties are expected to identify the issues in broad terms at the magistrates’ sending hearings the Crown Court will deal with the issues at the PTPH
The prosecutor’s statutory duty to disclose unused material to the accused is triggered by:​
* a plea of not guilty in the magistrates’ court;
* the sending for trial at the Crown Court;​

How well did you know this?
1
Not at all
2
3
4
5
Perfectly
21
Q

Carla is due to face trial for one count of robbery, at the arraignment Carla refuses to answer when she is asked to enter a plea.

What position should the court take?

The court should deem that Carla has entered a guilty plea

The court should deem that Carla has entered a not guilty plea

The court should hold Carla in contempt of court for not answering when being asked to enter a plea.

The court should deem that Carla is not fit to plead

A

The court should deem that Carla has entered a not guilty plea

Correct – if a defendant remains silent on arraignment then the court should deem that they have entered a not –guilty plea and set the matter for trial. Please review your materials on arraignment and what happens if a defendant does not answer when asked to enter a plea.

How well did you know this?
1
Not at all
2
3
4
5
Perfectly
22
Q

Pheobe and Ellie are charged with a series of thefts. It is alleged that one acts as a lookout whilst the other enters a shop and steals items of value. They take it in turns to act as lookout and enter the shop to steal.

You act for the prosecution and are responsible for drafting the indictment. Which of the following correctly reflects the position with the indictment?

There should be two indictments, one for each defendant, as you always need a separate indictment for separate defendants.

There should be one indictment with one count which deals with both defendants as the offences are founded on the same facts.

There should be two indictments, one for each defendant, as it would not be in the interests of justice for the defendants to be tried together.

There should be one indictment with two counts, one for each defendant as you always need a separate count for separate defendants.

A

There should be one indictment with one count which deals with both defendants as the offences are founded on the same facts.

Correct – Two defendants should be entered in the same count on the same indictment where the offence arises from the same facts or is part of the same series of offending – both are relevant here.

How well did you know this?
1
Not at all
2
3
4
5
Perfectly
23
Q

Alan and Gary are charged with a burglary of a dwelling, it is alleged that they committed the offence together. Alan is 45 years old and has lots of previous convictions for burglary. Gary is 23 and this is the first time that he has appeared before the courts.

You represent Gary who asks you if his case can be tried separately from Alan. Which of the following is the best advice for Gary?

The court will only order separate trials if it is of the opinion that separate trials are required in the interests of justice.

The court is not able to order separate trials in cases where the defendants are charged with the same count in circumstances where the offence arises out of the same set of facts.

The court will only order separate trials if it is of the opinion that hearing the cases together would cause prejudice or embarrassment to the accused and separate trials are required in the interests of justice.

The court will only order separate trials if it is of the opinion that hearing the cases together would cause prejudice to the case of the accused and separate trials are required in the interests of justice.

A

The court will only order separate trials if it is of the opinion that hearing the cases together would cause prejudice or embarrassment to the accused and separate trials are required in the interests of justice.

Correct – this is the correct test for the judge to apply when considering whether to hold separate trials for two defendants.

How well did you know this?
1
Not at all
2
3
4
5
Perfectly
24
Q

Question 1
A man has been arrested on Tuesday at 10.00 hrs on suspicion of common assault (a
summary offence) and taken to the local police station by the arresting officer. He arrives at
10.15 hrs and the custody officer authorises his detention to obtain evidence by questioning
him at 10.30 hrs.
Which of the following best describes the maximum period of time this man can be
detained before charge?
A He can be detained up until an inspector conducts a review of his detention which must
be by 16.30 hrs on Tuesday.
B He can be detained up until an inspector conducts a review of his detention which must
be by 19.30 hrs on Tuesday.
C He can be detained up until 10.00 hrs on Wednesday.
D He can be detained up until 10.15 hrs on Wednesday.
E He can be detained up until 10.30 hrs on Wednesday.

A

Answer
Option D is the best answer. The ‘custody clock’ as opposed to the ‘review clock’ starts from
the time the man arrives at the police station, which here is 10.15 hrs on the Tuesday (not
the time of his arrest away from the police station, so option C is wrong, nor from the time
detention is authorised, so option E is also wrong). According to s 41, the maximum period
of detention before charge is 24 hours (note this cannot be extended as common assault is
a summary- only offence) so he can only be detained up until 10.15 hrs on the Wednesday.
Option A correctly sets out the time by which the man’s detention must be initially reviewed
by an inspector, but the question asks for the maximum period of detention, not when the
first review must be conducted. Option B is wrong because this also refers to a review, and
in any event, it gives the wrong time period (9 hours) for the first review to be conducted.

25
Q

Question 2
A woman has been arrested and taken to the police station. When she is given her rights,
she requests legal advice, but she subsequently changes her mind when she learns the
police are ready to interview her and there is a delay in her solicitor attending the police
station. The custody officer authorises the interview to proceed in the absence of the
solicitor and makes an entry to this effect in the custody record. The woman also confirms
her change of mind in writing by signing an entry to this effect in the custody record.
Can the interview lawfully proceed on this basis?
A Yes, because the right to legal advice is an ongoing right, the suspect is entitled to
change her mind at any stage during her detention.
B No, because written authority can only be given by an officer not below the rank of
inspector.
C Yes, because the custody officer has authorised this and made a record of the
authorisation in the custody record.
D No, because written authority can only be given by an officer not below the rank of
superintendent.
E No, because once a suspect has requested legal advice the interview cannot proceed
without the solicitor’s attendance.

A

Answer
Option B is the best answer. Whilst a suspect can change her mind about wanting legal
advice (so option E is wrong), this must be authorised in writing by an officer not below the
rank of inspector (so options C and D are also wrong). Whilst option A is correct in that the
right to legal advice is an ongoing right, and the suspect is entitled to change her mind at
any stage during her detention, there are some additional safeguards that must be followed
for this to be done correctly. These safeguards are actually very detailed (see the key points
below), but option B is the best answer because this correctly identifies the role played by an
officer not below the rank of inspector, provided:
*
this officer speaks to the suspect to enquire about the reasons for their change of mind,
and makes reasonable efforts to contact the solicitor;
*
the suspect’s reason for the change of mind and the outcome of the efforts to contact the
solicitor are recorded in the custody record;
*
the suspect confirms in writing that they want the interview to proceed without first
obtaining legal advice by signing an entry to this effect in the custody record; and
*
the officer of the rank of inspector or above is satisfied that it is proper for the interview to
proceed in these circumstances and gives authority in writing for the interview to proceed.

26
Q

Question 3
A solicitor is acting for a client at the police station when it becomes apparent that there is
a conflict of interest with an existing client the solicitor is already representing.
Assuming the solicitor will withdraw from acting for the client, what should she now say
to the custody officer?
A That she no longer wants to represent the client.
B That she can no longer represent the client because there is a conflict of interest with
an existing client.
C That she can no longer represent the client and the custody officer should speak to the
client to find out why.
D That she can no longer represent the client but someone else from her firm will be
attending to represent the client.
E That she is no longer able to represent the client for professional reasons.

A

Answer
Option E is the best answer because all the solicitor can tell the custody officer is that
she can no longer act (for professional reasons) – she must not disclose the reason why.
Arguably she could just tell the custody officer that she is no longer able to act, but some
practitioners would say it would be discourteous not to explain that it is for professional
reasons. Option A is not the best answer because it is misleading for the solicitor to say
that she no longer wants to represent the client. The reason she can no longer represent
the client is because of her professional obligations to this client and her existing client
rather than because she no longer wants to do this. If the solicitor told the custody officer
why she was no longer able to act (other than for professional reasons) this would be a
breach of the ongoing duty of confidentiality owed to the client under para 6.3 of the SRA
Code of Conduct, so option B is wrong. It would also be wrong to tell the custody officer
to speak to the client about why the solicitor can no longer act, so option C is also wrong.
Option D is wrong because if the solicitor has a conflict with this client and an existing
client, this will also prohibit anyone else in the firm from representing the client.

27
Q

Question 1
A man has been arrested on suspicion of burglary. A witness claims to have seen the man
climbing out of a window of the burgled premises at the time of the burglary. The witness
claims she recognised the man because they were at school together. During interview, the
man denies any involvement in the burglary and claims the witness is mistaken. He also
claims the witness could not have recognised him on the basis that he is known to her as
they were not in the same class or year group at school and he left school six years ago.
Which of the following statements best describes whether the police will now be
required to hold an identification procedure?
A The holding of an identification procedure will be required because the man claims the
witness is mistaken and disputes that he is known to the witness.
B The police may hold an identification procedure if they believe it will help the
investigation into this offence.
C The police may hold an identification procedure if the officer in charge of the
investigation considers it would be useful.
D An identification procedure would not be necessary because the witness would
inevitably pick out the man.
E An identification procedure would serve no useful purpose as it is not disputed that the
man is already known to the witness.

A

Answer
Option A is the best answer. An identification would be mandatory in this case according
to Code D, para 3.12. There is a witness who has purported to recognise the man and
the man disputes being the person the witness claims to have seen, so an identification
procedure shall be held unless it is not practicable or would serve no useful purpose in
proving or disproving whether the man was involved in committing the offence. The man
disputes that he is known to the witness because they were not in the same year group at
school and also because of the passage of time since they were at school. This scenario
is therefore very similar to R v Harris (2003) where the CA stated that an identification
procedure was required in such circumstances. Options B and C are not the best answers
because although an eyewitness identification procedure may also be held if the officer in
charge of the investigation considers it would be useful (Code D, para 3.13), this would not
apply where the requirement to hold the procedure is already caught by para 3.12. Option
D is wrong because it is not at all inevitable that the witness would pick out the man since
the man has proper grounds to dispute that he is known to her. Option E is not the best
answer because it is disputed that the man is already known to the witness.

28
Q

Question 2
Two brothers have been arrested on suspicion of affray. The police have a witness to
the incident who believes he would be able to identify the two offenders. Both brothers
deny being involved in the incident and both claim they were elsewhere at the time.
Because the two brothers are roughly of similar appearance, the police hold a video
identification procedure with the images of the two brothers and 10 images of other
people who all resemble the suspects in age, height, general appearance and position
in life. The investigating officer, who is an officer of inspector rank, conducts the video
identification procedure.
Has the video identification procedure been properly conducted?
A Yes, because the correct number of images have been used and an officer not below
the rank of inspector has conducted the procedure.
B No, because two separate procedures should have been used with one of the suspect’s
video image and eight other images used for each procedure.
C Yes, because the other images are of people who all resemble the suspects in age,
height, general appearance and position in life.
D No, because the investigating officer must not be involved in the conduct of the
identification procedure.
E No, because 12 other images should have been used and the investigating officer must
not be involved in the identification procedure.

A

Answer
Option E is the best answer. The video identification procedure has not been conducted
lawfully in accordance with Annex A to Code D of PACE 1984 because 12 other images (not
10) should have been used and the investigating officer must not be involved in the conduct
of the procedure, so option A is wrong. Where two suspects of roughly similar appearance
are shown in the same images, they may be shown together with at least 12 other people
(Code D, Annex A, para 2), so although separate procedures would usually be held, option
B is not the best answer. Although option C is correct about the other images all resembling
the suspects in age, height, general appearance and position in life, it fails to explain why
the procedure is still not conducted correctly. Option D is partially correct, because the
investigating officer must not be involved in the conduct of the identification procedure, but
it is not the best answer, because it fails to mention that the wrong number of images (10
rather than 12) have been used.

29
Q

Question 3
A street robbery takes place and the victim provides a first description of the robber. The
victim does not know who the robber was and so she is shown photographs, in batches of
12, of people who fit this description. The victim makes a positive identification of a man
who is then arrested on suspicion of committing the robbery. The man is interviewed and
denies any involvement, claiming that he was elsewhere at the time of the robbery.
What are the police now required to do?
A The victim has already made a positive identification so the man should now be
charged.
B The victim should now be asked to take part in an identification procedure.
C The police will be required to hold an identification procedure because the officer in
charge of the investigation will consider it to be useful.
D The police should not have shown the victim photographs in batches of 12 so they will
not be able to proceed further against the man.
E The man should now be compelled to take part in an identification procedure.

A

Answer
Option B is the best answer. The victim has made an identification from viewing the
photographs and should now be asked to take part in an identification procedure (Code
D, Annex E, para 6). Option A is wrong because a positive identification has not yet been
made. Option C is not the best answer because although this may be one of the reasons
for conducting an identification procedure the victim must first be asked to take part in such
a procedure and in the circumstances of this case, para D 3.12 will be the reason why such
a procedure is required. Option D is wrong because the victim must be shown at least 12
photographs at a time (Code D, Annex E, para 4). Option E is wrong because the suspect
cannot be compelled to take part in such a procedure.

30
Q

Question 1
A defendant has been jointly charged with an offence of affray. At his first court
appearance in the magistrates’ court his solicitor advises him on plea and trial venue
before his case is called on. The defendant intends to indicate a not guilty plea and
decides to consent to summary trial if he is given the choice.
Which of the following best describes whether the defendant’s trial will take place in
the magistrates’ court?
A If both defendants consent to summary trial, the trial must take place in the
magistrates’ court.
B If the defendant consents to summary trial, the trial is likely to take place in the
magistrates’ court regardless of the other defendant’s decision.
C If the magistrates decide the offence to be more suitable for summary trial and both
defendants consent to summary, the trial will take place in the magistrates’ court.
D If the defendant consents to summary trial, the trial is likely to take place in the
magistrates’ court even if the other defendant elects trial on indictment.
E If the magistrates decide the offence to be more suitable for trial on indictment and
the other defendant also consents to summary trial, the trial will take place in the
magistrates’ court.

A

Answer
Option C is the best answer. CrimPR, r 9.2(6)(a) provides that where the court is dealing
with two or more defendants charged with the same offence, if one of the defendants
elects trial in the Crown Court, all of the defendants will be sent to the Crown Court for their
joint trial regardless of the other defendant(s)’s decision on venue. The trial will therefore
only take place in the magistrates’ court if the magistrates decide the offence to be more
suitable for summary trial and both defendants consent to summary trial. Option A is not
the best answer because it fails to deal first with the magistrates accepting jurisdiction.
Options B and D are wrong because both defendants have to consent to summary trial.
Option E is wrong because if the magistrates decide the offence to be more suitable for
trial on indictment, the case must be sent to the Crown Court for trial and the defendants
will not get a choice.

31
Q

Question 2
A man has been charged with an offence of rape and sexual assault in relation to the
same complainant. Both are alleged to have occurred within a few days of each other. The
man intends to plead not guilty to both charges and wants to know which court or courts
will deal with these cases.
Which of the following best describes the advice the man should be given?
A Both cases are likely to be tried in the Crown Court as rape is triable only on indictment.
B The rape charge will be dealt with in the Crown Court and the sexual assault charge
may be dealt with in the Crown Court if the magistrates decide their powers of
punishment are inadequate to deal with it.
C The rape charge will be dealt with in the Crown Court and the sexual assault charge
will only be dealt with in the Crown Court if the man elects trial on indictment.
D Both cases will be tried in the Crown Court as both charges relate to indictable
offences.
E Both cases will be tried in the Crown Court as the sexual assault charge is related to
the rape charge.

A

Answer
Option E is the best answer. The either- way offence (sexual assault) relates to an offence
triable only on indictment (rape) because they are alleged to have taken place over
a similar time period and both relate to the same complainant (CDA 1998, s 50A(3)
(a)). Consequently, the sexual assault charge will be sent to the Crown Court without an
allocation hearing by virtue of s 50A CDA 1998. So, option A is wrong because both cases
will be tried in the Crown Court, as opposed to being likely to be tried there. Options B and
C are wrong because there will be no allocation hearing. Option D is not the best answer
because although both offences are indictable offences, the reason they will be both dealt
with in the Crown Court is because the either- way offence is related to the offence that can
only be tried on indictment.

32
Q

Question 3
A woman appears in the magistrates’ court charged with an offence of assault occasioning
actual bodily harm and an offence of theft. At the plea before venue hearing she indicates
a not guilty plea to the assault matter and a guilty plea in relation to the theft matter.
The magistrates accept jurisdiction to deal with the assault, but the woman elects trial on
indictment.
Which court will sentence the woman for the offence of theft?
A The magistrates’ court will sentence the woman for the offence of theft as they have
already accepted jurisdiction to deal with the assault matter.
B The magistrates’ court will have a choice to either sentence the woman for the offence
of theft or to commit her to the Crown Court to be sentenced there.
C The woman will be able to choose which court she is sentenced by.
D The Crown Court will sentence the woman after her trial for the assault matter has
been concluded.
E The Crown Court will sentence the woman before her trial for the assault matter
takes place

A

Answer
Option B is the best answer. In this situation, the magistrates have a choice as to what to do
with the offence to which the defendant has pleaded guilty. They may either sentence the
defendant themselves or commit the defendant to the Crown Court for sentence. This will often
depend on whether the two offences are linked or not. If they are not, the magistrates’ court
may be more likely to sentence the offender, whereas if they are linked, the magistrates may
be more likely to send them both up to the Crown Court to be dealt with. Option A is therefore
wrong as accepting jurisdiction to deal with the assault matter will not necessarily have any
bearing on their decision to sentence for the theft. Option C is wrong, because a defendant
will never be able to choose which court sentences her for an either- way offence. This decision
will always be for the magistrates’ court to take. Options D and E are wrong because although
the theft offence could be committed to the Crown Court for sentence, this will not necessarily
be the case.

33
Q

Question 1
A solicitor is representing a defendant at his first appearance in court. The defendant has
been charged with inflicting grievous bodily harm. It is alleged he punched the victim,
fracturing his jaw. The defendant claims he was acting in self- defence. The defendant
does not know the victim. The defendant has no previous convictions for violence. His last
conviction was 18 months ago when he received a sentence of imprisonment of six months,
suspended for two years, for burglary. The defendant has three previous convictions
for failing to surrender to custody for a range of dishonesty offences. The defendant is
presently of no fixed abode.
Which of the following best explains the ground on which the prosecution is likely to
object to bail being granted to the defendant?
A There are substantial grounds to believe that the defendant will fail to surrender to
custody.
B The defendant may commit an offence whilst on bail given his antecedent history.
C The nature and seriousness of the offence with which the defendant has been charged.
D There are substantial grounds to believe the defendant will interfere with a key
prosecution witness.
E The defendant’s character, antecedents, associations and community ties.

A

Answer
Option A is the best answer. This is the only ground on these facts for refusal of bail given
that the defendant is presently subject to a suspended sentence of imprisonment which is
likely to be activated if he is convicted. Moreover, he appears to have poor community ties
as we are told he is of no fixed abode and he also has a number of previous convictions
for absconding. All of the other options are either not grounds for refusing bail or do not
apply to these facts. To refuse bail on the ground of committing further offences, there must be ‘substantial grounds to believe’ that he will commit offences whilst on bail. The fact
that he ‘may’ is not sufficient and in any event, there is no evidence that he has committed
offences whilst on bail in the past, so option B is not the best answer. Option C is wrong
because it is not a ground for refusal of bail at all; it is a factor for the court to take into
account when considering the grounds, as is option E. Option D is not the best answer
because there are no substantial grounds to believe that the client will interfere with a
prosecution witness. We are told that the defendant does not know the victim, nor is there
any suggestion that he has made threats to interfere with this witness or has ever done so
in the past (see Bail Act 1976, Sch 1).

34
Q

Question 2
A solicitor attends the cells in the magistrates’ court to speak to his client who has been
remanded in custody on his first appearance in court. The defendant’s case has been sent
to the Crown Court for a plea and trial preparation hearing (PTPH), and the defendant
wants to know whether he can make a further bail application in the magistrates’ court.
Can the defendant make a further bail application in the magistrates’ court?
A Yes, because he is entitled to make a further bail application relying on the same facts
and arguments as before.
B Yes, because he is entitled to make a further bail application but only if he can refer to
‘new argument’.
C Yes, because he is entitled to make two full bail applications before the
magistrates’ court.
D No, because his case has now been sent to the Crown Court and so his best option is
to appeal against the bail decision to the Crown Court.
E No, because his case has now been transferred to the Crown Court and so he can now
only apply for bail at the PTPH.

A

Answer
Option D is the best answer. At the first hearing after the hearing at which the magistrates
refused to grant bail, the defendant’s solicitor is allowed to make a full application for bail
using any argument as to fact or law, even if they used the same arguments in the first
unsuccessful bail application. However, options A and C would not be available because
we are told that the defendant’s case has now been sent to the Crown Court and so the
magistrates’ court will no longer have jurisdiction to hear a further bail application. Option
B is wrong because the requirement for new argument only applies after two full bail
applications have been made (Bail Act 1976, Sch 1, Pt IIA). Although option E is technically
correct, it will be much quicker to make a bail appeal before a judge in chambers in the
Crown Court rather than waiting some time for the PTPH in the Crown Court.

35
Q

Question 3
A man has been charged with an offence of burglary and bailed to attend the magistrates’
court in two weeks’ time. The police attach a bail condition which requires the man to
report to his local police station on a daily basis between the hours of 5 pm and 7 pm. The
man obtains employment which requires him to work away from home and he forgets to
report to his local police station between the allotted times on two consecutive days.
What will happen to the man as a result of breaching his bail condition?
A The man is likely to be arrested for breaching his bail condition, but he has a
reasonable excuse for breaching it if he can provide evidence of his present
employment status.
B The man is likely to be arrested for breaching his bail condition and bailed to attend
the magistrates’ court to answer for this breach.
C The man is likely to be arrested for breaching his bail condition and will be detained in
police custody and must then be brought before the magistrates’ court within 24 hours
although he does not commit an offence by breaching this condition.
D The man will be given a formal warning for breaching his bail condition and any further
breach will result in his arrest and production before the magistrates’ court to answer
the breach.
E The man will be arrested for breaching his bail condition and will be detained in police
custody and must then be brought before the magistrates’ court within 24 hours. He
also commits an offence by breaching this condition.

A

Answer
Option C is the best answer. Although breaching a bail condition (whether imposed by
the police or the court) does not amount to a criminal offence, it will almost certainly result
in his arrest and he will then be detained in police custody and must be brought before
the magistrates’ court within 24 hours. The magistrates’ court will then decide whether to
remand the man in custody, or whether to grant bail with or without conditions pending his
next substantive hearing.
Option A is wrong because this would not amount to a reasonable excuse. Option B
is wrong because the man would be unlikely to be bailed as a result of breaching this
condition. The police will usually let the magistrates’ court decide what to do in such
circumstances. Option D is wrong because breach of bail is not something that would
trigger the issuing of a formal warning. Option E is wrong because breaching this type of a
bail condition does not amount to a criminal offence.

36
Q

Question 1
A solicitor is representing a defendant at his first appearance in the magistrates’ court.
The defendant has been charged with offences of rape (an offence that can only be tried
on indictment) and sexual assault (an either- way offence). It is alleged that he sexually
assaulted the victim before he then raped her the following week. The defendant intends to
plead not guilty to both allegations.
Will both offences be sent to the Crown Court for trial?
A Yes, because the defendant is pleading not guilty to both allegations.
B No, because the sexual assault allegation can only be tried in the magistrates’ court.
C No, because the defendant may elect trial for both allegations in the magistrates’ court
or the Crown Court.
D Yes, because the sexual assault allegation is related to the rape allegation which must
be sent to the Crown Court for trial.
E Yes, because the sexual assault allegation is punishable with imprisonment and is
related to the rape allegation which must be sent to the Crown Court for trial.

A

Answer
Option D is the best answer. This is because rape is an offence triable only on indictment and
sexual assault is an either- way offence. Where an adult appears before a magistrates’ court
charged with an offence triable only on indictment, the court must send the defendant to the
Crown Court for trial pursuant to s 51(1) of the CDA 1998 for that offence; and for any either-
way offence (or summary offence) with which they are charged which fulfils the ‘requisite
conditions’. Here the ‘requisite conditions’ are that the either- way offence appears to the court
to be related to the offence triable only on indictment. Given that both allegations relate to
the same victim and they are alleged to have taken place within a week of each other, they
will be regarded as related to each other. Although sexual assault is also an imprisonable
offence, option E is not the best answer because this is only a requirement for a summary- only
offence. Option A is not the best answer, because the defendant will not be asked to indicate
his plea to either of these matters in this case (he would have been asked to indicate his plea
to the allegation of sexual assault if he had not also been charged with rape, an offence
that can only be tried on indictment). Option B is not correct as sexual assault is an either-
way offence. Option C is not correct because rape is not an either- way offence and so he will
never be given the choice to elect where his trial takes place.

37
Q

Question 2
A defendant has been charged with an offence of burglary that took place at office
premises in the city centre. As part of the investigation the police have a witness statement
from a neighbour of the defendant claiming she saw the defendant in his back garden
at the time the burglary took place. However, she tells the police that she is not prepared
to give evidence about this as she does not like her neighbour. The prosecution does not
believe the neighbour is telling the truth and does not intend to call her as a witness at the
defendant’s trial in the magistrates’ court.
Which of the following best describes whether the prosecution will be required to
disclose the existence of the neighbour’s witness statement to the defence?
A The prosecution will be required to disclose this material as part of the initial details of
the prosecution case.
B The prosecution will not be required to disclose this witness statement because the trial
is taking place in the magistrates’ court and not the Crown Court.
C The prosecution will be required to disclose this witness statement because it provides
the defendant with an alibi defence.
D The prosecution will not be required to disclose this witness statement because it is
sensitive material and subject to public interest immunity as the neighbour does not
want to testify at trial.
E The prosecution will be required to disclose this witness statement because it might
reasonably be considered capable of undermining the case for the prosecution.

A

Answer
Option E is the best answer. Section 3 CPIA 1996 provides that such unused material must
be disclosed to the defence if it ‘might reasonably be considered capable of undermining
the case for the prosecution’. Arguably this witness statement might also reasonably be
considered capable of assisting the case for the accused, but we are not told what defence
the defendant is raising and, in any event, this is not one of the options available. Option
A is not correct because this witness statement will not be provided as IDPC, as we are told
the prosecution will not be calling this witness to give evidence. The obligation to disclose
unused prosecution material applies to trials in the magistrates’ court and Crown Court, so option B is not correct. Option C is not the best answer, because although this may support
the defendant’s defence of alibi, we do not know whether the defendant is raising such a
defence and in any event, this does not set out the test for disclosure in s 3. Option D is not
correct because a witness’s unwillingness to testify does not make it sensitive material that
may be eligible for a public interest immunity application.

38
Q

Question 3
A man is facing trial in the Crown Court for an offence of wounding. The prosecution has
provided disclosure of their non- sensitive, unused material which might reasonably be
considered capable of undermining their case or of assisting the case for the defendant.
What obligation, if any, is now on the defendant to make disclosure of his defence to
the prosecution?
A The defendant is required to serve a defence statement. This obligation will be
enforced by the court by drawing an adverse inference against the defendant if the
statement is not provided.
B The defendant may serve a defence statement but cannot be obliged to do so. Failure
to provide the statement means the prosecution will not be required to review their
initial disclosure and determine if there is any further relevant unused material in its
possession.
C The defendant is not required to serve a defence statement on the prosecution as this
would violate the defendant’s right to a fair trial and his presumption of innocence.
D The prosecution cannot insist on the defendant providing a defence statement, but
the court is likely to make an award of costs against the defendant for failing to
provide one.
E The defendant is not required to serve a defence statement but there are good tactical
reasons for serving one. This includes placing an obligation on the prosecution to then
provide disclosure of their sensitive, unused material.

A

Answer
Option A is the best answer. Section 5 of the CPIA 1996 sets out the obligation on the
defence to provide a defence statement. This is referred to as ‘compulsory disclosure’ where
a defendant is facing trial in the Crown Court (as opposed to the magistrates’ court where
there is no such obligation). This obligation is enforced in the Crown Court by the court
being able to draw an adverse inference against the defendant if a defence statement is
not provided.
Option B is wrong, although this would be correct if the man’s trial were taking place in
the magistrates’ court. Option C is wrong because the obligation to provide a defence
statement does not violate his right to a fair trial, nor his presumption of innocence. Option
D is correct to the extent that the prosecution cannot insist on the defendant providing a
defence statement, but the court would not make an award of costs against the defendant
for failing to do so. Option E is wrong because the defendant is required to serve this
statement. Moreover, this would not trigger an obligation on the prosecution to then
provide disclosure of their sensitive, unused material. Such material will never be disclosed
if the prosecution makes a successful public immunity interest application to withhold its
disclosure.

39
Q

A man has been arrested on suspicion of common assault. He is a professional football player earning £180,000 per year. It is alleged that the man punched a photographer who was trying to take a picture of him whilst he was having a drink with a woman in a bar. The man has never been in trouble with the police before and has requested that he is represented by the duty solicitor while at the police station.

Will the man be entitled to publicly funded legal representation in connection with the defence of his case?

A. The man will not be eligible for publicly funded legal representation by the duty solicitor at the police station because he will fail the means test.

B. The man will be eligible for publicly funded legal representation by the duty solicitor at the police station and at all hearings up to and including sentence.

C. The man will be eligible for publicly funded legal representation by the duty solicitor at the police station, limited to telephone advice only.

D. The man will be eligible for publicly funded legal representation by the duty solicitor at the police station. He will also be able to get a representation order in the Magistrates’ Court provided that he passes the interests of justice test, because there is no means test in the Magistrates’ Court.

E. The man will be eligible for publicly funded legal representation by the duty solicitor at the police station. He will not be able to get a representation order in the Magistrates’ Court, because even if he passes the interests of justice test, he will fail the means test which applies in the Magistrates’ Court.

A

E - The man will be eligible for publicly funded legal representation by the duty solicitor at the police station. He will not be able to get a representation order in the Magistrates’ Court, because even if he passes the interests of justice test, he will fail the means test which applies in the Magistrates’ Court.

40
Q

A man is arrested at 11pm on a Wednesday, under suspicion of causing grievous bodily harm with intent to a woman. He is brought to the police station arriving at 11.30pm and his detention is authorised at 11.45pm.

The police investigations are ongoing. There are searches to be undertaken, witnesses to be located and medical evidence to be obtained. There will be a delay in obtaining the medical evidence as the woman is undergoing extensive surgery. The investigating officers anticipate that they will need to use the maximum powers of detention available to the police before charge and may need to seek a warrant of further detention from the Magistrates’ Court.

Until what time can the man be detained before charge without a warrant of further detention?

A. 11.30pm Thursday.

B. 11.45pm Thursday.

C. 11am Friday.

D. 11.30am Friday.

E. 11.45am Friday.

A

D - 11.30am Friday.

41
Q

The police are called to a bar in relation to an allegation of an assault. The attending police officers speak to a man who is bleeding from a cut to the face. A woman is arrested at the bar on suspicion of assaulting the man occasioning him actual bodily harm. The arresting officer notices blood on the woman’s hand.

The woman is taken to a police station. Her detention is authorised and she is subsequently interviewed under caution in the presence of her solicitor. On the advice of her solicitor, the woman exercises her right to silence in the interview and does not give an explanation for the blood on her hand.

The interviewing officer warns the woman that an assault is being investigated and that she is being asked to account for the blood on her hand as it is believed that the blood may be present because she committed the assault. She is reminded that the interview is being recorded and the record may be given in evidence at a court hearing, and that an inference may later be drawn by a court if she fails to account for the blood during the interview. The woman continues to exercise her right to silence.

The woman is charged with assaulting the man occasioning him actual bodily harm. The woman pleads not guilty and refuses to give evidence at her trial.

Can the trial court be invited to draw an adverse inference from the woman’s failure to account for the blood on her hand in her interview under caution?

A. No, because since the woman is not giving evidence at her trial, she is not relying upon a fact in her defence which she did not mention in her interview under caution.

B. No, because the woman has a right to silence in her interview under caution.

C. No, because the woman was advised by her solicitor to exercise her right to silence in her interview under caution.

D. Yes, because of the warning given to the woman in her interview under caution.

E. Yes, because it would have been reasonable for the woman to explain the blood on her hand in her interview under caution.

A

D - Yes, because of the warning given to the woman in her interview under caution.

42
Q

Eva is appearing at a magistrates’ court charged with robbery of a petrol station clerk. She denies the charge.

At Eva’s first appearance at a magistrates’ court, what advice will you give her about where her case will be tried?

It must be tried in the Crown Court

It may be tried in a magistrates’ court, if the magistrates accept jurisdiction

It may be tried in the Crown Court

It must be tried in a magistrates’ court

A

It must be tried in the Crown Court

Correct. Robbery is an indictable only offence which must be tried in the Crown Court.
The other options were incorrect because: * either-way offences ‘may’ be tried in a magistrates’ court or the Crown Court; and * summary only offences must be tried in a magistrates’ court.

43
Q

What is the overriding objective of the Criminal Procedure Rules (CrimPR)?

The overriding objective of the Criminal Procedure Rules is that criminal cases be dealt with fairly.

The overriding objective of the Criminal Procedure Rules is that criminal cases acquit the innocent and convict the guilty.

The overriding objective of the Criminal Procedure Rules is that criminal cases be dealt with justly.

The overriding objective of the Criminal Procedure Rules is that criminal cases be dealt with efficiently and expeditiously.

A

The overriding objective of the Criminal Procedure Rules is that criminal cases be dealt with justly.

This is the correct answer, as it is the wording in CrimPR 1.1(1). The other answers may sound plausible as CrimPR 1.1(2) explains that dealing with a case justly includes: - (a) acquitting the innocent and convicting the guilty; - (b) dealing with the prosecution and the defence fairly; and - (e) dealing with the case efficiently and expeditiously.

44
Q

An officer wishes to arrest a suspect without a warrant.

Which of the following is a ground for arrest?

The suspect’s address needs to be ascertained

The suspect is about to cause physical injury to himself or another person

The suspect is in the act of committing an offence

The suspect’s name needs to be ascertained

A

The suspect is in the act of committing an offence

This is the correct answer. Section 24(1)(b) PACE 1984 states that ‘[a] constable may arrest without a warrant anyone who is in the act of committing an offence’. Other grounds for arrest without a warrant are: anyone who is about to commit an offence, anyone a constable has reasonable grounds for suspecting to be about to commit an offence and anyone a constable has reasonable grounds for suspecting to be committing an offence.
The remainder of the answers are a selection of the reasons for arrest, found in s.24(5) rather than grounds of arrest. The constable has reasonable grounds for believing that it is necessary to arrest the person to, for example: enable the name of the person in question to be ascertained, enable the person’s address to be ascertained or to prevent the person causing physical injury to himself or another person.

45
Q

Fred, a well-known criminal, is suspected of being an accomplice to robbery. Based on a tip off from a member of the public, the police arrest Fred at a local betting shop. As the police officers arrived Fred said, ‘you must be here about me helping rob that woman’. One police officer arrested Fred and another said ‘you are under arrest’.

Was the arrest lawful?

The arrest was unlawful as this is not the procedure to be followed for serious, indictable offences.

The arrest was unlawful as one officer performed the physical arrest and the other told Fred he was under arrest.

The arrest was unlawful as no ground for his arrest was given.

The arrest was lawful as Fred was told he was under arrest and the ground was obvious from his statement.

A

The arrest was unlawful as no ground for his arrest was given.

This is the correct answer. A suspect must be told he is under arrest and the ground for the arrest, s.28(3) PACE 1984.
There is no problem with one officer performing the physical arrest and another officer advising the suspect that the arrest is taking place.
There is no separate procedure for arresting a person suspected of an indictable offence.

46
Q

Which of the following powers requires an application by the police to the magistrates’ court?

Power to detain for up to 96 hours

Power to arrest without a warrant

The power to detain for up to 24 hours.

Power to detain for up to 36 hours

A

Power to detain for up to 96 hours

This is the correct answer. In order to extend the period of detention without charge beyond 36 hours, an application must be made by the police to the magistrates’ court for a warrant for further detention.
The police have the power to detain for up to 36 hours if authorised by an officer who holds the rank of superintendent or above who believes that the investigation is being conducted diligently and expeditiously, s.42(1) PACE 1984.
The powers to stop and arrest without a warrant do not require an application to the magistrates’ court.

47
Q

Billy has been arrested on suspicion of handling stolen goods. It was alleged by an anonymous source that he was storing several valuable items at his home address that had been stolen during a recent burglary. The burglary was committed by a known associate of Billy’s. Billy was arrested at his home address and during a search of his spare bedroom, police officers seized several items that had been reported as stolen. He was taken to the police station for questioning.

What are Billy’s rights upon detention?

Billy is entitled to free and independent legal advice, a private consultation with a solicitor and to have someone informed of his arrest. If he chooses to access legal advice, then he is not entitled to consult the Codes of Practice.

Billy is entitled to free and independent legal advice, a private consultation with a solicitor, to have someone informed of his arrest and to consult the Codes of Practice.

Billy is entitled to free and independent legal advice, a private consultation with a solicitor, a telephone call to inform someone of his arrest and to consult the Codes of Practice.

Billy is entitled to free and independent legal advice, a private consultation with a solicitor and to have someone informed of his arrest. Billy can only receive legal advice after he has signed the custody record to indicate that he has read the Codes of Practice.

A

Billy is entitled to free and independent legal advice, a private consultation with a solicitor, to have someone informed of his arrest and to consult the Codes of Practice.

Correct. The custody officer must make sure that Billy, as an arrested person is told clearly about the following continuing rights which may be exercised at any stage during the period in custody: * the right to consult privately with a solicitor/ legal representative and that free independent legal advice is available; * the right to have someone informed of his arrest; and * the right to consult the Codes of Practice. Legal advice has no bearing on the right to consult the Codes and vice versa. They are separate and distinct rights.
The custody record will record that these rights have been given and any response made by the suspect (e.g. the name of the solicitor or person to be notified).
A telephone call is no longer a right of a detained person under the Codes of Practice however Billy does have a right to have someone informed of his arrest.

48
Q

The police have arrested Joanne on suspicion of theft. Just prior to her interview, she is shouting that she is the Queen and hurling herself against the wall of her cell. The police suspect Joanne may be mentally disordered.

Who can act as an appropriate adult for Joanne?

Joanne’s solicitor

A social worker Joanne has made admissions to

A police officer or police employee

Someone who has experience of dealing with such persons

A

Someone who has experience of dealing with such persons

This is the correct answer, although Joanne’s wishes should be respected where practicable. See PACE Code C, Note for Guidance 1D for further details.
Your materials on suspect’s rights in detention deal with this point, in particular the right to an appropriate adult where required.
A police officer or police employee cannot act as her appropriate adult, see PACE Code C, para 1.7(b).
A social worker Joanne has made admissions to cannot act as her appropriate adult, see PACE Code C, Note for Guidance 1C.
Joanne’s solicitor may cannot act as her appropriate adult, see PACE Code C, Note for Guidance 1F.

49
Q

Ismail (aged 14) is a looked after child. He is in the care of the local authority as Ismail’s mother has severe learning difficulties with the mental capacity of a 9 year old and is therefore unable to look after him. Ismail has been arrested on suspicion of shoplifting. He is taken to the local police station and booked in by the custody officer. The police want to interview him as soon as possible but are required under PACE 1984 to secure the attendance of an appropriate adult.

Who should the police contact to attend the police station and act in Ismail’s best interests as his appropriate adult?

Ismail’s solicitor

Ismail’s brother, aged 17

Ismail’s mother

Ismail’s social worker

A

Ismail’s social worker

Correct. Ismail is a ‘looked after child’ and, as such his social worker can act as an appropriate adult.
Technically a parent could act as an appropriate adult but in this case Ismail’s mother doesn’t have the mental capacity to act in this role (Code C, para 1.7).
A solicitor should not act as an appropriate adult if attending a police station on a suspect’s behalf (Code C, Note for Guidance 1F).
An appropriate adult must be a responsible adult aged 18 or over and Ismail’s brother is 17 years old (Code C, para 1.7(a)).

50
Q

Veronica is charged with an offence of criminal damage. The allegation is that she deliberately scratched a car with her keys, causing £500 of damage. Your instructing solicitor asks whether there is a time limit within which the prosecution must begin her case.

What advice will you give your instructing solicitor?

The prosecution needs to commence proceedings within 12 months of the date of the alleged offence

The prosecution needs to commence proceedings within 6 months of the date of the alleged offence

The prosecution has no time limit within which it needs to commence proceedings

The prosecution needs to commence proceedings within 3 months of the date of the alleged offence

A

The prosecution needs to commence proceedings within 6 months of the date of the alleged offence

Correct. Section 127(1) Magistrates’ Courts Act 1980 provides that where the alleged offence is summary only (as in this case of criminal damage £5,000 or under), a magistrates’ court shall not try an information or hear a complaint unless the information was laid or the complaint made within six months of the date of the alleged offence.

51
Q

You are instructed to represent Oscar. He was arrested following an incident where a significant drugs deal had ‘turned bad’ and a fight broke out. During the fight, it is said that Oscar produced a firearm and shot at Craig. Craig was not hit. Oscar accepts that he was dealing drugs to Craig, but states that Craig attacked him first, and he acted in self-defence. He denies that any gun was used or fired. Oscar is charged with supply of drugs (which he admits) and possession of a firearm with intent to endanger life, an indictable-only offence (which he does not).

Assume that Oscar was remanded in custody at the first hearing. How many days can he be held in custody until his trial?

182 days from when Oscar is sent to the Crown Court minus any time during which he was in the custody of the magistrates’ court.

56 days from the date of the first hearing in the magistrates’ court.

182 days from when Oscar is sent to the Crown Court.

182 days from the date of the first hearing in the magistrates’ court.

A

182 days from when Oscar is sent to the Crown Court minus any time during which he was in the custody of the magistrates’ court.

Correct. This answer states the custody time limit between sending an indictable only matter to the Crown Court (Crime and Disorder Act 1998 s.51) and the start of the trial (as per Regulation 5(6B) Prosecution of Offences (Custody Time Limits) Regulations 1987).
The other options may seem plausible but are incorrect because they:
· took the date of the first hearing in the magistrates’ court as when the custody time limit starts instead of from when Oscar is sent to the Crown Court;
· failed to account for any time during which Oscar was in the custody of the magistrates’ court; or
· stated Oscar could be held in custody for 56 days from the date of the first hearing in the magistrates’ court- this is the custody time limit between first hearing and summary trial, which cannot apply to this indictable only offence.

52
Q

Hannah, aged 25, is charged with murder, an indictable only offence. The allegation is that she murdered her husband by poisoning him. She intends to enter a plea of not guilty.

Where will Hannah have her first hearing and what will happen at it?

Hannah will have her first hearing in a magistrates’ court. Hannah’s hearing must be via live link given the seriousness of the allegation.

Hannah will have her first hearing in a magistrates’ court. Hannah will be entitled to initial details of the prosecution case.

Hannah will have her first hearing in the Crown Court. Hannah will be asked to enter a plea.

Hannah will have her first hearing in a magistrates’ court. Hannah will be asked to enter a plea.

A

Hannah will have her first hearing in a magistrates’ court. Hannah will be entitled to initial details of the prosecution case.

Correct. All adult defendants have their first hearing in a magistrates’ court. Hannah will be entitled to initial details of the prosecution case, according to CrimPR part 8.
The other options are incorrect because:
· the severity of the allegation does not determine the use of a live link; and
· Hannah cannot be asked to enter a plea in the first hearing for an indictable only offence, Crime and Disorder Act 1998, s51(1).

53
Q

Helen is charged with theft. The allegation is that she stole a dress worth £199 from a local boutique. She denies the charge.

Who decides where Helen’s case will be heard?

If the prosecution decide Helen’s case should be heard in the Crown Court then she and the magistrates have no choice in the matter.

If Helen decides her case should be heard in a magistrates’ court then she can insist on the matter being heard there and the magistrates have no choice in the matter.

If the magistrates decide Helen’s case should be heard in the Crown Court then she and the prosecution will have no choice in the matter.

If the magistrates decide Helen’s case should be heard in a magistrates’ court then she and the prosecution have no choice in the matter

A

If Helen decides her case should be heard in a magistrates’ court then she can insist on the matter being heard there and the magistrates have no choice in the matter.

Correct. As this is an offence of ‘low value shoplifting’ it is treated as summary only unless Helen, the defendant, elects Crown Court trial.
The other options were incorrect because:
· the prosecution do not have the power to decide where Helen’s case will be heard;
· the magistrates have no power to send this case to the Crown Court; and
· Helen has a choice where the matter will be heard.

54
Q

You represent Thomas who is jointly charged with Ryan with theft of some copper wires from a building site. Both defendants wish to plead not guilty. Thomas indicates that he wishes to elect summary trial but he believes that his co-defendant, appearing at court on the same occasion, wishes to have his trial in the Crown Court. Thomas is very anxious for his case to remain in a magistrates’ court.

On which basis is the court and the parties most likely to proceed in relation to election?

Defence counsel for Thomas seeks an adjournment, because if the pleas of Ryan and Thomas are taken separately on different days, Ryan can elect Crown Court trial, and Thomas can have his trial in a magistrates’ court as he would not be appearing with Ryan.

The court finds that the case is suitable for summary jurisdiction. Once Ryan elects Crown Court trial, defence counsel can make representations to the court that Thomas’s case remains in a magistrates’ court.

The court makes an enquiry of the parties on their intentions as to election prior to the mode of trial procedure. The bench makes their decision as to whether to accept jurisdiction or not.

The court finds that the case is suitable for summary jurisdiction. Once Thomas consents to summary trial, Ryan no longer has the right to elect Crown Court trial.

A

The court makes an enquiry of the parties on their intentions as to election prior to the mode of trial procedure. The bench makes their decision as to whether to accept jurisdiction or not.

Correct. Pursuant to CrimPR 9.2(6)(b) the court may ask the accused questions to help it decide in what order to deal with the defendants and include questions in relation to their intention to elect Crown Court trial.
The other options were incorrect because:
· once Ryan, charged with the same offence, has elected Crown Court trial, Thomas must be sent forthwith, despite any wish to remain in a magistrates’ court;
· Ryan and Thomas must be advised that if one of them is sent for Crown Court trial, either because the magistrates have declined jurisdiction or because one of the co-defendants has elected Crown Court trial, they will both be jointly sent for trial, CrimPR 9.2(6)(a); and
· the occasion upon which the plea is taken is irrelevant. As both defendants have appeared at court together, they remain ‘jointly charged’ regardless of whether Thomas’s plea is dealt with on a subsequent date. In this scenario the two accused have appeared at court on the same occasion and an adjournment would still require that the court ‘must’ send him to join Ryan at the Crown Court.

55
Q

Iqbal is charged with assault occasioning actual bodily harm (‘ABH’). The alleged facts are that he assaulted his wife, Rita, during a domestic argument, by hitting her in the face and head, causing significant bruising. Neighbours called the police after hearing screaming from Rita. Iqbal left the house, got into his car and drove away. He was stopped by police. It transpired that Iqbal was disqualified from driving some months previously. He was arrested for ABH and driving whilst disqualified. Iqbal was remanded into custody and appeared before the magistrates the following morning.

What is the most appropriate advice for you to give Iqbal about where and when these matters will be heard?

The driving whilst disqualified matter and the ABH matter will be heard in the Crown Court and dealt with together.

The driving whilst disqualified matter and the ABH matter will be heard in a magistrates’ court, but they will be dealt with separately.

The driving whilst disqualified matter can be adjourned to be heard by a magistrates’ court after the ABH matter is sent to the Crown Court and dealt with.

The driving whilst disqualified matter can be sent to the Crown Court for trial on indictment alongside the ABH matter, if Iqbal elects Crown Court trial or a magistrates’ court declines jurisdiction.

A

The driving whilst disqualified matter can be sent to the Crown Court for trial on indictment alongside the ABH matter, if Iqbal elects Crown Court trial or a magistrates’ court declines jurisdiction.

Correct. Driving whilst disqualified is an offence listed in s.40 Criminal Justice Act 1988 and therefore can be sent for trial on indictment with the ABH.
The other options were not the most appropriate advice because:
· as an either-way offence the ABH matter will not necessarily be heard in the Crown Court;
· the ABH may be dealt with in a magistrates’ court or the Crown court; and
· on the facts it is unlikely that the two matters will be severed, as they arise out of the same facts.

56
Q

Amanda is charged with an offence of common assault. The allegation is that she assaulted a fellow member of staff. Amanda denies the allegation.

At Amanda’s first appearance at a magistrates’ court, what advice will you give her about where her case will be tried?

It may be tried in a magistrates’ court, but if she is found guilty she can be committed by the magistrates to the Crown Court for sentencing

It must be tried in a magistrates’ court

It may be tried in a magistrates’ court if the magistrates accept jurisdiction, but if she is found guilty she cannot be committed by the magistrates to the Crown Court for sentencing

It must be tried in the Crown Court

A

It must be tried in a magistrates’ court

Correct. As a summary only offence, this common assault case must be tried in a magistrates’ court.
The other options were incorrect because as a summary only offence:
· it will not be tried in the Crown Court;
· the magistrates do not choose to accept jurisdiction in a summary only offence; and
· the magistrates cannot commit Amanda for sentencing.
There are certain circumstances, where a defendant is charged with an either-way or indictable only matter and a summary only matter, the summary only matter can be heard in the Crown Court, however, this is not one of those cases.

57
Q

When can an application under s.8 Criminal Procedure and Investigations Act 1996 for disclosure of prosecution material be made?

After initial disclosure of unused prosecution material, but before service of the defence statement.

After initial details of the prosecution case have been served, but before initial disclosure of unused prosecution material.

At the hearing at which the case is sent to the Crown Court.

After initial disclosure of unused prosecution material and service of the defence statement.

A

After initial disclosure of unused prosecution material and service of the defence statement.

Correct. An application under s.8 Criminal Procedure and Investigations Act 1996 for disclosure of prosecution material be made after initial disclosure of unused prosecution material and service of the defence statement.
The other options, while they may sound plausible are incorrect because essentially, they are too soon. A defence statement needs to be file and served before an application for specific disclosure can be made. Service of the defence statement will happen after initial disclosure of unused prosecution material and the hearing at which the case is sent to the Crown Court.

58
Q

Which of the following correctly explains when the defence statement must be served?

In the magistrates’ court, a defence statement must be served within 28 days of the prosecution complying, or purporting to comply, with its duty of initial disclosure of unused material.

In the magistrates’ court, a defence statement must be served within 7 days of the prosecution complying, or purporting to comply, with its duty of initial disclosure of unused material.

In the Crown Court, a defence statement must be served within 28 days of the prosecution complying, or purporting to comply, with its duty of initial disclosure of unused material.

In the Crown Court, a defence statement must be served within 14 days of the prosecution complying, or purporting to comply, with its duty of initial disclosure of unused material.

A

In the Crown Court, a defence statement must be served within 28 days of the prosecution complying, or purporting to comply, with its duty of initial disclosure of unused material.

Correct. In the Crown Court, a defence statement is compulsory, so once initial disclosure of unused material has been made, a defence statement must be filed and served by the defence within 28 days.
In the magistrates’ court a defence statement is not compulsory, but if a defendant chooses to serve a defence statement, standard directions provide the defendant must do so within 14 days of the prosecution complying, or purporting to comply, with its initial duty of disclosure.

59
Q

Harry is charged with actual bodily harm and his case has been adjourned for a summary trial.

Which of the following statements, regarding disclosure, is correct?

Harry is entitled to disclosure of unused material and is entitled to make an application for disclosure of prosecution material.

Harry must file and serve a defence statement in order to make an application for disclosure of prosecution material.

Harry is not entitled to disclosure of unused material, as he is facing summary trial.

Harry must file and serve a defence statement in order to obtain disclosure of unused material.

A

Harry must file and serve a defence statement in order to make an application for disclosure of prosecution material.

Correct
Correct. This answer correctly reflects s.8 Criminal Procedure and Investigations Act 1996 on applications for specific disclosure of prosecution material.
The other options are incorrect because:
· Harry is entitled to disclosure of unused material (provided that it meets the test for disclosure)- this is not dependent on filing and serving a defence statement; and
· Harry must file and serve a defence statement before an application for specific disclosure can be made.